IMMUNE_Ch. 35, 36, 37, 38.

Ace your homework & exams now with Quizwiz!

During a mumps outbreak at a local school, a patient, who is a school teacher, is exposed. She has previously been immunized for mumps. What type of immunity does she possess? A) Acquired immunity B) Natural immunity C) Phagocytic immunity D) Humoral immunity

A (Feedback: Acquired immunity usually develops as a result of prior exposure to an antigen, often through immunization. When the body is attacked by bacteria, viruses, or other pathogens, it has three means of defense. The first line of defense, the phagocytic immune response, involves the WBCs that have the ability to ingest foreign particles. A second protective response is the humoral immune response, which begins when the B lymphocytes transform themselves into plasma cells that manufacture antibodies. The natural immune response system is rapid, nonspecific immunity present at birth.)

An infection control nurse is presenting an inservice reviewing the immune response. The nurse describes the clumping effect that occurs when an antibody acts like a cross-link between two antigens. What process is the nurse explaining? A) Agglutination B) Cellular immune response C) Humoral response D) Phagocytic immune response

A (Feedback: Agglutination refers to the clumping effect occurring when an antibody acts as a cross-link between two antigens. This takes place within the context of the humoral immune response, but is not synonymous with it. Cellular immune response, the immune system's third line of defense, involves the attack of pathogens by T-cells. The phagocytic immune response, or immune response, is the system's first line of defense, involving white blood cells that have the ability to ingest foreign particles.)

The nurse is providing care for a patient who has multiple sclerosis. The nurse recognizes the autoimmune etiology of this disease and the potential benefits of what treatment? A) Stem cell transplantation B) Serial immunizations C) Immunosuppression D) Genetic engineering

A (Feedback: Clinical trials using stem cells are under way in patients with a variety of disorders having an autoimmune component, including multiple sclerosis. Immunizations and genetic engineering are not used to treat multiple sclerosis. Immunosuppression would exacerbate symptoms of MS)

A patient with a history of dermatitis takes corticosteroids on a regular basis. The nurse should assess the patient for which of the following complications of therapy? A) Immunosuppression B) Agranulocytosis C) Anemia D) Thrombocytopenia

A (Feedback: Corticosteroids such as prednisone can cause immunosuppression. Corticosteroids do not typically cause agranulocytosis, anemia, or low platelet counts.)

A gardener sustained a deep laceration while working and requires sutures. The patient is asked about the date of her last tetanus shot, which is over 10 years ago. Based on this information, the patient will receive a tetanus immunization. The tetanus injection will allow for the release of what? A) Antibodies B) Antigens C) Cytokines D) Phagocytes

A (Feedback: Immunizations activate the humoral immune response, culminating in antibody production. Antigens are the substances that induce the production of antibodies. Immunizations do not prompt cytokine or phagocyte production)

A patient has undergone treatment for septic shock and received high doses of numerous antibiotics during the course of treatment. When planning the patient's subsequent care, the nurse should be aware of what potential effect on the patient's immune function? A) Bone marrow suppression B) Uncontrolled apoptosis C) Thymus atrophy D) Lymphoma

A (Feedback: Large doses of antibiotics can precipitate bone marrow suppression, affecting immune function. Antibiotics are not noted to cause apoptosis, thymus atrophy, or lymphoma.)

A patient is vigilant in her efforts to "take good care of herself" but is frustrated by her recent history of upper respiratory infections and influenza. What aspect of the patient's lifestyle may have a negative effect on immune response? A) The patient works out at the gym twice daily. B) The patient does not eat red meats. C) The patient takes over-the-counter dietary supplements. D) The patient sleeps approximately 6 hours each night.

A (Feedback: Rigorous exercise or competitive exercise—usually considered a positive lifestyle factor—can be a physiologic stressor and cause negative effects on immune response. The patient's habits around diet and sleep do not present obvious threats to immune function.)

A woman has been diagnosed with breast cancer and is being treated aggressively with a chemotherapeutic regimen. As a result of this regimen, she has an inability to fight infection due to the fact that her bone marrow is unable to produce a sufficient amount of what? A) Lymphocytes B) Cytoblasts C) Antibodies D) Capillaries

A (Feedback: The white blood cells involved in immunity (including lymphocytes) are produced in the bone marrow. Cytoblasts are the protoplasm of the cell outside the nucleus. Antibodies are produced by lymphocytes, but not in the bone marrow. Capillaries are small blood vessels)

A patient's exposure to which of the following microorganisms is most likely to trigger a cellular response? A) Herpes simplex B) Staphylococcus aureus C) Pseudomonas aeruginosa D) Beta hemolytic Streptococcus

A (Feedback: Viral, rather than bacterial antigens, induce a cellular response.)

A patient with multiple food and environmental allergies tells the nurse that he is frustrated and angry about having to be so watchful all the time and wonders if it is really worth it. What would be the nurse's best response? A) "I can only imagine how you feel. Would you like to talk about it?" B) "Let's find a quiet spot and I'll teach you a few coping strategies." C) "That's the same way that most patients who have a chronic illness feel." D) "Do you think that maybe you could be managing things more efficiently?"

A) "I can only imagine how you feel. Would you like to talk about it?" To assist the patient in adjusting to these modifications, the nurse must have an appreciation of the difficulties encountered by the patient. The patient is encouraged to verbalize feelings and concerns in a supportive environment and to identify strategies to deal with them effectively. The nurse should not suggest that the patient has been mismanaging his health problem and the nurse should not make comparisons with other patients. Further assessment should precede educational interventions.

A nurse has asked the nurse educator if there is any way to predict the severity of a patient's anaphylactic reaction. What would be the nurse's best response? A) "The faster the onset of symptoms, the more severe the reaction." B) "The reaction will be about one-third more severe than the patient's last reaction to the same antigen." C) "There is no way to gauge the severity of a patient's anaphylaxis, even if it has occurred repeatedly in the past." D) "The reaction will generally be slightly less severe than the last reaction to the same antigen."

A) "The faster the onset of symptoms, the more severe the reaction." The time from exposure to the antigen to onset of symptoms is a good indicator of the severity of the reaction: the faster the onset, the more severe the reaction. None of the other statements is an accurate description of the course of anaphylactic reactions.

An office worker takes a cupcake that contains peanut butter. He begins wheezing, with an inspiratory stridor and air hunger and the occupational health nurse is called to the office. The nurse should recognize that the worker is likely suffering from which type of hypersensitivity? A) Anaphylactic (type 1) B) Cytotoxic (type II) C) Immune complex (type III) D) Delayed-type (type IV)

A) Anaphylactic (type 1) The most severe form of a hypersensitivity reaction is anaphylaxis. An unanticipated severe allergic reaction that is often explosive in onset, anaphylaxis is characterized by edema in many tissues, including the larynx, and is often accompanied by hypotension, bronchospasm, and cardiovascular collapse in severe cases. Type II, or cytotoxic, hypersensitivity occurs when the system mistakenly identifies a normal constituent of the body as foreign. Immune complex (type III) hypersensitivity involves immune complexes formed when antigens bind to antibodies. Type III is associated with systemic lupus erythematosus, rheumatoid arthritis, certain types of nephritis, and bacterial endocarditis. Delayed-type (type IV), also known as cellular hypersensitivity, occurs 24 to 72 hours after exposure to an allergen.

The nurse is providing care for a patient who has experienced a type I hypersensitivity reaction. What condition is an example of such a reaction? A) Anaphylactic reaction after a bee sting B) Skin reaction resulting from adhesive tape C) Myasthenia gravis D) Rheumatoid arthritis

A) Anaphylactic reaction after a bee sting Anaphylactic (type I) hypersensitivity is an immediate reaction mediated by IgE antibodies and requires previous exposure to the specific antigen. Skin reactions are more commonly type IV and myasthenia gravis is thought to be a type II reaction. Rheumatoid arthritis is not a type I hypersensitivity reaction.

A school nurse is caring for a child who appears to be having an allergic response. What should be the initial action of the school nurse? A) Assess for signs and symptoms of anaphylaxis. B) Assess for erythema and urticaria. C) Administer an OTC antihistamine. D) Administer epinephrine.

A) Assess for signs and symptoms of anaphylaxis. If a patient is experiencing an allergic response, the nurse's initial action is to assess the patient for signs and symptoms of anaphylaxis. Erythema and urticaria may be present, but these are not the most significant or most common signs of anaphylaxis. Assessment must precede interventions, such as administering an antihistamine. Epinephrine is indicated in the treatment of anaphylaxis, not for every allergic reaction.

A patient who has been newly diagnosed with systemic lupus erythematosus (SLE) has been admitted to the medical unit. Which of the following nursing diagnoses is the most plausible inclusion in the plan of care? A) Fatigue Related to Anemia B) Risk for Ineffective Tissue Perfusion Related to Venous Thromboembolism C) Acute Confusion Related to Increased Serum Ammonia Levels D) Risk for Ineffective Tissue Perfusion Related to Increased Hematocrit

A) Fatigue Related to Anemia (Patients with SLE nearly always experience fatigue, which is partly attributable to anemia. Ammonia levels are not affected and hematocrit is typically low, not high. VTE is not one of the central complications of SLE.)

A patient has been brought to the emergency department by EMS after being found unresponsive. Rapid assessment reveals anaphylaxis as a potential cause of the patient's condition. The care team should attempt to assess for what potential causes of anaphylaxis? Select all that apply. A) Foods B) Medications C) Insect stings D) Autoimmunity E) Environmental pollutants

A) Foods B) Medications C) Insect stings Substances that most commonly cause anaphylaxis include foods, medications, insect stings, and latex. Pollutants do not commonly cause anaphylaxis and autoimmune processes are more closely associated with types II and III hypersensitivities.

A nurse is working with a patient with rheumatic disease who is being treated with salicylate therapy. What statement would indicate that the patient is experiencing adverse effects of this drug? A) I have this ringing in my ears that just wont go away. B) I feel so foggy in the mornings and it takes me so long to wake up. C) When I eat a meal thats high in fat, I get really nauseous. D) I seem to have lost my appetite, which is unusual for me.

A) I have this ringing in my ears that just wont go away. Tinnitus is associated with salicylate therapy. Salicylates do not normally cause drowsiness, intolerance of high-fat meals, or anorexia.)

A patient has been admitted to the emergency department with signs of anaphylaxis following a bee sting. The nurse knows that if this is a true allergic reaction the patient will present with what alteration in laboratory values? A) Increased eosinophils B) Increased neutrophils C) Increased serum albumin D) Decreased blood glucose

A) Increased eosinophils Higher percentages of eosinophils are considered moderate to severe eosinophilia. Moderate eosinophilia is defined as 15% to 40% eosinophils and is found in patients with allergic disorders. Hypersensitivity does not result in hypoglycemia or increased albumin and neutrophil counts.

A nurse is planning the care of a patient who has a long history of chronic pain, which has only recently been diagnosed as fibromyalgia. What nursing diagnosis is most likely to apply to this womans care needs? A) Ineffective Role Performance Related to Pain B) Risk for Impaired Skin Integrity Related to Myalgia C) Risk for Infection Related to Tissue Alterations D) Unilateral Neglect Related to Neuropathic Pain

A) Ineffective Role Performance Related to Pain (Typically, patients with fibromyalgia have endured their symptoms for a long period of time. The neuropathic pain accompanying FM can often impair a patient's ability to perform normal roles and functions. Skin integrity is unaffected and the disease has no associated infection risk. Activity limitations may result in neglect, but not of a unilateral nature.)

A patient has a diagnosis of rheumatoid arthritis and the primary care provider has now prescribed cyclophosphamide (Cytoxan). The nurses subsequent assessments should address what potential adverse effect? A) Infection B) Acute confusion C) Sedation D) Malignant hyperthermia

A) Infection (Feedback:When administering immunosuppressives such as Cytoxan, the nurse should be alert to manifestations of bone marrow suppression and infection. Confusion and sedation are atypical adverse effects. Malignant hyperthermia is a surgical complication and not a possible adverse effect.)

A patient is undergoing diagnostic testing to determine the etiology of recent joint pain. The patient asks the nurse about the difference between osteoarthritis (OA) and rheumatoid arthritis (RA). What is the best response by the nurse? A) OA is a considered a noninflammatory joint disease. RA is characterized by inflamed, swollen joints. B) OA and RA are very similar. OA affects the smaller joints such as the fingers, and RA affects the larger, weight-bearing joints like the knees. C) OA originates with an infection. RA is a result of your bodys cells attacking one another. D) OA is associated with impaired immune function; RA is a consequence of physical damage.

A) OA is a considered a noninflammatory joint disease. RA is characterized by inflamed, swollen joints. (OA is a degenerative arthritis with a noninflammatory etiology, characterized by the loss of cartilage on the articular surfaces of weight-bearing joints, with spur development. RA is characterized by inflammation of synovial membranes and surrounding structures. The diseases are not distinguished by the joints affected and neither has an infectious etiology.)

A patient has been admitted to a medical unit with a diagnosis of polymyalgia rheumatica (PMR). The nurse should be aware of what aspects of PMR? Select all that apply. A) PMR has an association with the genetic marker HLA-DR4. B) Immunoglobulin deposits occur in PMR. C) PMR is considered to be a wear-and-tear disease. D) Foods high in purines exacerbate the biochemical processes that occur in PMR. E) PMR occurs predominately in Caucasians.

A) PMR has an association with the genetic marker HLA-DR4. B) Immunoglobulin deposits occur in PMR. E) PMR occurs predominately in Caucasians. (The underlying mechanism involved with polymyalgia rheumatica is unknown. This disease occurs predominately in Caucasians and often in first-degree relatives. An association with the genetic marker HLA-DR4 suggests a familial predisposition. Immunoglobulin deposits in the walls of inflamed temporal arteries also suggest an autoimmune process. Purines are unrelated and it is not a result of physical degeneration.)

The nurse is preparing to care for a patient who has scleroderma. The nurse refers to resources that describe CREST syndrome. Which of the following is a component of CREST syndrome? A) Raynauds phenomenon B) Thyroid dysfunction C) Esophageal varices D) Osteopenia

A) Raynauds phenomenon (The "R" in CREST stands for Raynaud's phenomenon. Thyroid dysfunction, esophageal varices, and osteopenia are not associated with scleroderma.)

After the completion of testing, a child's allergies have been attributed to her family's cat. When introducing the family to the principles of avoidance therapy, the nurse should promote what action? A) Removing the cat from the family's home B) Administering OTC antihistamines to the child regularly C) Keeping the cat restricted from the child's bedroom D) Maximizing airflow in the house

A) Removing the cat from the family's home In avoidance therapy, every attempt is made to remove the allergens that act as precipitating factors. Fully removing the cat from the environment is preferable to just keeping the cat out of the child's bedroom. Avoidance therapy does not involve improving airflow or using antihistamines.

A patients decreased mobility is ultimately the result of an autoimmune reaction originating in the synovial tissue, which caused the formation of pannus. This patient has been diagnosed with what health problem? A) Rheumatoid arthritis (RA) B) Systemic lupus erythematosus C) Osteoporosis D) Polymyositis

A) Rheumatoid arthritis (RA) In RA, the autoimmune reaction results in phagocytosis, producing enzymes within the joint that break down collagen, cause edema and proliferation of the synovial membrane, and ultimately form pannus. Pannus destroys cartilage and bone. SLE, osteoporosis, and polymyositis do not involve pannus formation.

The nurse is planning the care of a patient who has a diagnosis of atopic dermatitis, which commonly affects both of her hands and forearms. What risk nursing diagnosis should the nurse include in the patient's care plan? A) Risk for Disturbed Body Image Related to Skin Lesions B) Risk for Disuse Syndrome Related to Dermatitis C) Risk for Ineffective Role Performance Related to Dermatitis D) Risk for Self-Care Deficit Related to Skin Lesions

A) Risk for Disturbed Body Image Related to Skin Lesions The highly visible skin lesions associated with atopic dermatitis constitute a risk for disturbed body image. This may culminate in ineffective role performance, but this is not likely the case for the majority of patients. Dermatitis is unlikely to cause a disuse syndrome or self-care deficit.

A nurse is caring for a 78-year-old patient with a history of osteoarthritis (OA). When planning the patients care, what goal should the nurse include? A) The patient will express satisfaction with her ability to perform ADLs. B) The patient will recover from OA within 6 months. C) The patient will adhere to the prescribed plan of care. D) The patient will deny signs or symptoms of OA.

A) The patient will express satisfaction with her ability to perform ADLs. Pain management and optimal functional ability are major goals of nursing interventions for OA. Cure is not a possibility and it is unrealistic to expect a complete absence of signs and symptoms. Adherence to the plan of care is highly beneficial, but this is not the priority goal of care.

A patient has sought care, stating that she developed hives overnight. The nurse's inspection confirms the presence of urticaria. What type of allergic hypersensitivity reaction has the patient developed? A) Type I B) Type II C) Type III D) Type IV

A) Type I Urticaria (hives) is a type I hypersensitive allergic reaction

A 5-year-old boy has been diagnosed with a severe food allergy. What is an important parameter to address when educating the parents of this child about his allergy and care? A) Wear a medical identification bracelet. B) Know how to use the antihistamine pen. C) Know how to give injections of lidocaine. D) Avoid live attenuated vaccinations.

A) Wear a medical identification bracelet. The nurse also advises the patient to wear a medical identification bracelet or to carry emergency equipment at all times. Patients and their families do not carry antihistamine pens, they carry epinephrine pens. Lidocaine is not self-administered to treat allergies. The patient may safely be vaccinated.

A patient with an exacerbation of systemic lupus erythematosus (SLE) has been hospitalized on the medical unit. The nurse observes that the patient expresses angerand irritation when her call bell isnt answered immediately. What would be the most appropriate response? A) You seem like youre feeling angry. Is that something that we could talk about? B) Try to remember that stress can make your symptoms worse. C) Would you like to talk about the problem with the nursing supervisor? D) I can see youre angry. Ill come back when youve calmed down.

A) You seem like youre feeling angry. Is that something that we could talk about? (The changes and the unpredictable course of SLE necessitate expert assessment skills and nursing care, as well as sensitivity to the psychological reactions of the patient. Offering to listen to the patient express anger can help the nurse and the patient understand its cause and begin to deal with it. Although stress can exacerbate the symptoms of SLE, telling the patient to calm down doesn't acknowledge her feelings. Ignoring the patient's feelings suggests that the nurse has no interest in what the patient has said. Offering to get the nursing supervisor also does not acknowledge the patient's feelings.)

The nurse is caring for a patient who has been admitted for the treatment of AIDS. In the morning, the patient tells the nurse that he experienced night sweats and recently coughed up some blood. What is the nurse's most appropriate action? A) Assess the patient for additional signs and symptoms of Kaposi's sarcoma. B) Review the patient's most recent viral load and CD4+ count. C) Place the patient on respiratory isolation and inform the physician. D) Perform oral suctioning to reduce the patient's risk for aspiration.

Ans: C Feedback: These signs and symptoms are suggestive of tuberculosis, not Kaposi's sarcoma; prompt assessment and treatment is necessary. There is no indication of a need for oral suctioning and the patient's blood work will not reflect the onset of this opportunistic infection.

A hospital patient is immunocompromised because of stage 3 HIV infection and the physician has ordered a chest radiograph. How should the nurse most safely facilitate the test? A) Arrange for a portable x-ray machine to be used. B) Have the patient wear a mask to the x-ray department. C) Ensure that the radiology department has been disinfected prior to the test. D) Send the patient to the x-ray department, and have the staff in the department wear masks.

Ans: A Feedback: A patient who is immunocompromised is at an increased risk of contracting nosocomial infections due to suppressed immunity. The safest way the test can be facilitated is to have a portable x-ray machine in the patient's room. This confers more protection than disinfecting the radiology department or using masks.

A patient with HIV infection has begun experiencing severe diarrhea. What is the most appropriate nursing intervention to help alleviate the diarrhea? A) Administer antidiarrheal medications on a scheduled basis, as ordered. B) Encourage the patient to eat three balanced meals and a snack at bedtime. C) Increase the patient's oral fluid intake. D) Encourage the patient to increase his or her activity level.

Ans: A Feedback: Administering antidiarrheal agents on a regular schedule may be more beneficial than administering them on an as-needed basis, provided the patient's diarrhea is not caused by an infectious microorganism. Increased oral fluid may exacerbate diarrhea; IV fluid replacement is often indicated. Small, more frequent meals may be beneficial, and it is unrealistic to increase activity while the patient has frequent diarrhea.

A nurse is planning the care of a patient with AIDS who is admitted to the unit withPneumocystis pneumonia (PCP). Which nursing diagnosis has the highest priority for this patient? A) Ineffective Airway Clearance B) Impaired Oral Mucous Membranes C) Imbalanced Nutrition: Less than Body Requirements D) Activity Intolerance

Ans: A Feedback: Although all these nursing diagnoses are appropriate for a patient with AIDS, Ineffective Airway Clearance is the priority nursing diagnosis for the patient with Pneumocystis pneumonia (PCP). Airway and breathing take top priority over the other listed concerns.

A patient with HIV has a nursing diagnosis of Risk for Impaired Skin Integrity. What nursing intervention best addresses this risk? A) Utilize a pressure-reducing mattress. B) Limit the patient's physical activity. C) Apply antibiotic ointment to dependent skin surfaces. D) Avoid contact with synthetic fabrics.

Ans: A Feedback: Devices such as alternating-pressure mattresses and low-air-loss beds are used to prevent skin breakdown. Activity should be promoted, not limited, and contact with synthetic fabrics does not necessary threaten skin integrity. Antibiotic ointments are not normally used unless there is a break in the skin surface.

Since the emergence of HIV/AIDS, there have been significant changes in epidemiologic trends. Members of what group currently have the greatest risk of contracting HIV? A) Gay, bisexual, and other men who have sex with men B) Recreational drug users C) Blood transfusion recipients D) Health care providers

Ans: A Feedback: Gay, bisexual, and other men who have sex with men remain the population most affected by HIV and account for 2% of the population but 61% of the new infections. This exceeds the incidence among drug users, health care workers, and transfusion recipients.

A clinic nurse is caring for a patient admitted with AIDS. The nurse has assessed that the patient is experiencing a progressive decline in cognitive, behavioral, and motor functions. The nurse recognizes that these symptoms are most likely related to the onset of what complication? A) HIV encephalopathy B) B-cell lymphoma C) Kaposi's sarcoma D) Wasting syndrome

Ans: A Feedback: HIV encephalopathy is a clinical syndrome characterized by a progressive decline in cognitive, behavioral, and motor functions. The other listed complications do not normally have cognitive and behavioral manifestations.

An HIV-infected patient presents at the clinic for a scheduled CD4+ count. The results of the test are 45 cells/mL, and the nurse recognizes the patient's increased risk for Mycobacterium aviumcomplex (MAC disease). The nurse should anticipate the administration of what drug? A) Azithromycin B) Vancomycin C) Levofloxacin D) Fluconazole

Ans: A Feedback: HIV-infected adults and adolescents should receive chemoprophylaxis against disseminatedMycobacterium avium complex (MAC disease) if they have a CD4+ count less than 50 cells/µL. Azithromycin (Zithromax) or clarithromycin (Biaxin) are the preferred prophylactic agents. Vancomycin, levofloxacin, and fluconazole are not prophylactic agents for MAC.

A nurse is addressing the incidence and prevalence of HIV infection among older adults. What principle should guide the nurse's choice of educational interventions? A) Many older adults do not see themselves as being at risk for HIV infection. B) Many older adults are not aware of the difference between HIV and AIDS. C) Older adults tend to have more sex partners than younger adults. D) Older adults have the highest incidence of intravenous drug use.

Ans: A Feedback: It is known that many older adults do not see themselves as being at risk for HIV infection. Knowledge of the relationship between HIV infection and AIDS is not known to affect the incidence of new cases. The statements about sex partners and IV drug use are untrue.

A patient who has AIDS is being treated in the hospital and admits to having periods of extreme anxiety. What would be the most appropriate nursing intervention? A) Teach the patient guided imagery. B) Give the patient more control of her antiretroviral regimen. C) Increase the patient's activity level. D) Collaborate with the patient's physician to obtain an order for hydromorphone.

Ans: A Feedback: Measures such as relaxation and guided imagery may be beneficial because they decrease anxiety, which contributes to weakness and fatigue. Increased activity may be of benefit, but for other patients this may exacerbate feelings of anxiety or loss. Granting the patient control has the potential to reduce anxiety, but the patient is not normally given unilateral control of the ART regimen. Hydromorphone is not used to treat anxiety.

A nurse is assessing the skin integrity of a patient who has AIDS. When performing this inspection, the nurse should prioritize assessment of what skin surfaces? A) Perianal region and oral mucosa B) Sacral region and lower abdomen C) Scalp and skin over the scapulae D) Axillae and upper thorax

Ans: A Feedback: The nurse should inspect all the patient's skin surfaces and mucous membranes, but the oral mucosa and perianal region are particularly vulnerable to skin breakdown and fungal infection.

The nurse's plan of care for a patient with stage 3 HIV addresses the diagnosis of Risk for Impaired Skin Integrity Related to Candidiasis. What nursing intervention best addresses this risk? A) Providing thorough oral care before and after meals B) Administering prophylactic antibiotics C) Promoting nutrition and adequate fluid intake D) Applying skin emollients as needed

Ans: A Feedback: Thorough mouth care has the potential to prevent or limit the severity of this infection. Antibiotics are irrelevant because of the fungal etiology. The patient requires adequate food and fluids, but these do not necessarily prevent candidiasis. Skin emollients are not appropriate because candidiasis is usually oral.

A nurse is performing the admission assessment of a patient who has AIDS. What components should the nurse include in this comprehensive assessment? Select all that apply. A) Current medication regimen B) Identification of patient's support system C) Immune system function D) Genetic risk factors for HIV E) History of sexual practices

Ans: A, B, C, E Feedback: Nursing assessment includes numerous focuses, including identification of medication use, support system, immune function and sexual history. HIV does not have a genetic component.

A nurse is completing a nutritional status of a patient who has been admitted with AIDS-related complications. What components should the nurse include in this assessment? Select all that apply. A) Serum albumin level B) Weight history C) White blood cell count D) Body mass index E) Blood urea nitrogen (BUN) level

Ans: A, B, D, E Feedback: Nutritional status is assessed by obtaining a dietary history and identifying factors that may interfere with oral intake, such as anorexia, nausea, vomiting, oral pain, or difficulty swallowing. In addition, the patient's ability to purchase and prepare food is assessed. Weight history (i.e., changes over time); anthropometric measurements; and blood urea nitrogen (BUN), serum protein, albumin, and transferrin levels provide objective measurements of nutritional status. White cell count is not a typical component of a nutritional assessment.

A patient is beginning an antiretroviral drug regimen shortly after being diagnosed with HIV. What nursing action is most likely to increase the likelihood of successful therapy? A) Promoting appropriate use of complementary therapies B) Addressing possible barriers to adherence C) Educating the patient about the pathophysiology of HIV D) Teaching the patient about the need for follow-up blood work

Ans: B Feedback: ART is highly dependent on adherence to treatment, and the nurse should proactively address this. Blood work is necessary, but this will not have a direct bearing on the success or failure of treatment. Complementary therapies are appropriate, but are not the main factor in successful treatment. The patient may or may not benefit from teaching about HIV pathophysiology.

A hospital nurse has experienced percutaneous exposure to an HIV-positive patient's blood as a result of a needlestick injury. The nurse has informed the supervisor and identified the patient. What action should the nurse take next? A) Flush the wound site with chlorhexidine. B) Report to the emergency department or employee health department. C) Apply a hydrocolloid dressing to the wound site. D) Follow up with the nurse's primary care provider.

Ans: B Feedback: After initiating the emergency reporting system, the nurse should report as quickly as possible to the employee health services, the emergency department, or other designated treatment facility. Flushing is recommended, but chlorhexidine is not used for this purpose. Applying a dressing is not recommended. Following up with the nurse's own primary care provider would require an unacceptable delay.

A nurse is performing an admission assessment on a patient with stage 3 HIV. After assessing the patient's gastrointestinal system and analyzing the data, what is most likely to be the priority nursing diagnosis? A) Acute Abdominal Pain B) Diarrhea C) Bowel Incontinence D) Constipation

Ans: B Feedback: Diarrhea is a problem in 50% to 60% of all AIDS patients. As such, this nursing diagnosis is more likely than abdominal pain, incontinence, or constipation, though none of these diagnoses is guaranteed not to apply.

A patient has come into contact with HIV. As a result, HIV glycoproteins have fused with the patient's CD4+ T-cell membranes. This process characterizes what phase in the HIV life cycle? A) Integration B) Attachment C) Cleavage D) Budding

Ans: B Feedback: During the process of attachment, glycoproteins of HIV bind with the host's uninfected CD4+ receptor and chemokine coreceptors, which results in fusion of HIV with the CD4+ T-cell membrane. Integration, cleavage, and budding are steps that are subsequent to this initial phase of the HIV life cycle.

A nurse is assessing a 28-year-old man with HIV who has been admitted with pneumonia. In assessing the patient, which of the following observations takes immediate priority? A) Oral temperature of 100°F B) Tachypnea and restlessness C) Frequent loose stools D) Weight loss of 1 pound since yesterday

Ans: B Feedback: In prioritizing care, the pneumonia would be assessed first by the nurse. Tachypnea and restlessness are symptoms of altered respiratory status and need immediate priority. Weight loss of 1 pound is probably fluid related; frequent loose stools would not take short-term precedence over a temperature or tachypnea and restlessness. An oral temperature of 100°F is not considered a fever and would not be the first issue addressed.

A patient with HIV will be receiving care in the home setting. What aspect of self-care should the nurse emphasize during discharge education? A) Appropriate use of prophylactic antibiotics B) Importance of personal hygiene C) Signs and symptoms of wasting syndrome D) Strategies for adjusting antiretroviral dosages

Ans: B Feedback: Infection control is of high importance in patients living with HIV, thus personal hygiene is paramount. This is a more important topic than signs and symptoms of one specific complication (wasting syndrome). Drug dosages should never be independently adjusted. Prophylactic antibiotics are not normally prescribed unless the patient's CD4 count is below 50.

A patient who has AIDS has been admitted for the treatment of Kaposi's sarcoma. What nursing diagnosis should the nurse associate with this complication of AIDS? A) Risk for Disuse Syndrome Related to Kaposi's Sarcoma B) Impaired Skin Integrity Related to Kaposi's Sarcoma C) Diarrhea Related to Kaposi's Sarcoma D) Impaired Swallowing Related to Kaposi's Sarcoma

Ans: B Feedback: Kaposi's sarcoma (KS) is a disease that involves the endothelial layer of blood and lymphatic vessels. This malignancy does not directly affect swallowing or bowel motility and it does not constitute a risk for disuse syndrome.

A patient is in the primary infection stage of HIV. What is true of this patient's current health status? A) The patient's HIV antibodies are successfully, but temporarily, killing the virus. B) The patient is infected with HIV but lacks HIV-specific antibodies. C) The patient's risk for opportunistic infections is at its peak. D) The patient may or may not develop long-standing HIV infection.

Ans: B Feedback: The period from infection with HIV to the development of HIV-specific antibodies is known as primary infection. The virus is not being eradicated and infection is certain. Opportunistic infections emerge much later in the course of the disease.

A patient with HIV is admitted to the hospital because of chronic severe diarrhea. The nurse caring for this patient should expect the physician to order what drug for the management of the patient's diarrhea? A) Zithromax B) Sandostatin C) Levaquin D) Biaxin

Ans: B Feedback: Therapy with octreotide acetate (Sandostatin), a synthetic analogue of somatostatin, has been shown to be effective in managing chronic severe diarrhea. Zithromax, Levaquin, and Biaxin are not used to treat chronic severe diarrhea.

A public health nurse is preparing an educational campaign to address a recent local increase in the incidence of HIV infection. The nurse should prioritize which of the following interventions? A) Lifestyle actions that improve immune function B) Educational programs that focus on control and prevention C) Appropriate use of standard precautions D) Screening programs for youth and young adults

Ans: B Feedback: Until an effective vaccine is developed, preventing HIV by eliminating and reducing risk behaviors is essential. Educational interventions are the primary means by which behaviors can be influenced. Screening is appropriate, but education is paramount. Enhancing immune function does not prevent HIV infection. Ineffective use of standard precautions apply to very few cases of HIV infection.

A nurse is working with a patient who was diagnosed with HIV several months earlier. The nurse should recognize that a patient with HIV is considered to have AIDS at the point when the CD4+ T-lymphocyte cell count drops below what threshold? A) 75 cells/mm3 of blood B) 200 cells/mm3 of blood C) 325 cells/mm3 of blood D) 450 cells/mm3 of blood

Ans: B Feedback: When CD4+ T-cell levels drop below 200 cells/mm3 of blood, the person is said to have AIDS.

A nurse is caring for a patient hospitalized with AIDS. A friend comes to visit the patient and privately asks the nurse about the risk of contracting HIV when visiting the patient. What is the nurse's best response? A) Do you think that you might already have HIV? B) Don't worry. Your immune system is likely very healthy. C) AIDS isn't transmitted by casual contact. D) You can't contract AIDS in a hospital setting.

Ans: C Feedback: AIDS is commonly transmitted by contact with blood and body fluids. Patients, family, and friends must be reassured that HIV is not spread through casual contact. A healthy immune system is not necessarily a protection against HIV. A hospital setting does not necessarily preclude HIV infection.

A nurse would identify that a colleague needs additional instruction on standard precautions when the colleague exhibits which of the following behaviors? A) The nurse wears face protection, gloves, and a gown when irrigating a wound. B) The nurse washes the hands with a waterless antiseptic agent after removing a pair of soiled gloves. C) The nurse puts on a second pair of gloves over soiled gloves while performing a bloody procedure. D) The nurse places a used needle and syringe in the puncture-resistant container without capping the needle.

Ans: C Feedback: Gloves must be changed after contact with materials that may contain high concentration of microorganisms, even when working with the same patient. Each of the other listed actions adheres to standard precautions.

A patient with AIDS is admitted to the hospital with AIDS-related wasting syndrome and AIDS-related anorexia. What drug has been found to promote significant weight gain in AIDS patients by increasing body fat stores? A) Advera B) Momordicacharantia C) Megestrol D) Ranitidine

Ans: C Feedback: Megestrol acetate (Megace), a synthetic oral progesterone preparation, promotes significant weight gain. In patients with HIV infection, it increases body weight primarily by increasing body fat stores. Advera is a nutritional supplement that has been developed specifically for people with HIV infection and AIDS. Momordicacharantia (bitter melon) is given as an enema and is part of alternative treatment for HIV/AIDS. Ranitidine prevents ulcers.

The nurse care plan for a patient with AIDS includes the diagnosis of Risk for Impaired Skin Integrity. What nursing intervention should be included in the plan of care? A) Maximize the patient's fluid intake. B) Provide total parenteral nutrition (TPN). C) Keep the patient's bed linens free of wrinkles. D) Provide the patient with snug clothing at all times.

Ans: C Feedback: Skin surfaces are protected from friction and rubbing by keeping bed linens free of wrinkles and avoiding tight or restrictive clothing. Fluid intake should be adequate, and must be monitored, but maximizing fluid intake is not a goal. TPN is a nutritional intervention of last resort.

A patient was tested for HIV using enzyme immunoassay (EIA) and results were positive. The nurse should expect the primary care provider to order what test to confirm the EIA test results? A) Another EIA test B) Viral load test C) Western blot test D) CD4/CD8 ratio

Ans: C Feedback: The Western blot test detects antibodies to HIV and is used to confirm the EIA test results. The viral load test measures HIV RNA in the plasma and is not used to confirm EIA test results, but instead to track the progression of the disease process. The CD4/CD8 ratio test evaluates the ratio of CD4 and CD8 cells but is not used to confirm results of EIA testing.

The mother of two young children has been diagnosed with HIV and expresses fear of dying. How should the nurse best respond to the patient? A) Would you like me to have the chaplain come speak with you? B) You'll learn much about the promise of a cure for HIV. C) Can you tell me what concerns you most about dying? D) You need to maintain hope because you may live for several years.

Ans: C Feedback: The nurse can help the patient verbalize feelings and identify resources for support. The nurse should respond with an open-ended question to help the patient to identify fears about being diagnosed with a life-threatening chronic illness. Immediate deferral to spiritual care is not a substitute for engaging with the patient. The nurse should attempt to foster hope, but not in a way that downplays the patient's expressed fears.

A patient with a recent diagnosis of HIV infection expresses an interest in exploring alternative and complementary therapies. How should the nurse best respond? A) Complementary therapies generally have not been approved, so patients are usually discouraged from using them. B) Researchers have not looked at the benefits of alternative therapy for patients with HIV, so we suggest that you stay away from these therapies until there is solid research data available. C) Many patients with HIV use some type of alternative therapy and, as with most health treatments, there are benefits and risks. D) You'll need to meet with your doctor to choose between an alternative approach to treatment and a medical approach.

Ans: C Feedback: The nurse should approach the topic of alternative or complementary therapies from an open-ended, supportive approach, emphasizing the need to communicate with care providers. Complementary therapies and medical treatment are not mutually exclusive, though some contraindications exist. Research supports the efficacy of some forms of complementary and alternative treatment.

A patient's primary infection with HIV has subsided and an equilibrium now exists between HIV levels and the patient's immune response. This physiologic state is known as which of the following? A) Static stage B) Latent stage C) Viral set point D) Window period

Ans: C Feedback: The remaining amount of virus in the body after primary infection is referred to as the viral set point, which results in a steady state of infection that lasts for years. This is not known as the static or latent stage. The window period is the time a person infected with HIV tests negative even though he or she is infected.

A patient has been diagnosed with AIDS complicated by chronic diarrhea. What nursing intervention would be appropriate for this patient? A) Position the patient in the high Fowler's position whenever possible. B) Temporarily eliminate animal protein from the patient's diet. C) Make sure the patient eats at least two servings of raw fruit each day. D) Obtain a stool culture to identify possible pathogens.

Ans: D Feedback: A stool culture should be obtained to determine the possible presence of microorganisms that cause diarrhea. Patients should generally avoid raw fruit when having diarrhea. There is no need to avoid animal protein or increase the height of the patient's bed.

A patient's current antiretroviral regimen includes nucleoside reverse transcriptase inhibitors (NRTIs). What dietary counseling will the nurse provide based on the patient's medication regimen? A) Avoid high-fat meals while taking this medication. B) Limit fluid intake to 2 liters a day. C) Limit sodium intake to 2 grams per day. D) Take this medication without regard to meals.

Ans: D Feedback: Many NRTIs exist, but all of them may be safely taken without regard to meals. Protein, fluid, and sodium restrictions play no role in relation to these drugs.

An 18-year-old pregnant female has tested positive for HIV and asks the nurse if her baby is going to be born with HIV. What is the nurse's best response? A) There is no way to know that for certain, but we do know that your baby has a one in four chance of being born with HIV. B) Your physician is likely the best one to ask that question. C) If the baby is HIV positive there is nothing that can be done until it is born, so try your best not to worry about it now. D) It's possible that your baby could contract HIV, either before, during, or after delivery.

Ans: D Feedback: Mother-to-child transmission of HIV-1 is possible and may occur in utero, at the time of delivery, or through breast-feeding. There is no evidence that the infant's risk is 25%. Deferral to the physician is not a substitute for responding appropriately to the patient's concern. Downplaying the patient's concerns is inappropriate.

A 16-year-old has come to the clinic and asks to talk to a nurse. The nurse asks the teen what she needs and the teen responds that she has become sexually active and is concerned about getting HIV. The teen asks the nurse what she can do keep from getting HIV. What would be the nurse's best response? A) There's no way to be sure you won't get HIV except to use condoms correctly. B) Only the correct use of a female condom protects against the transmission of HIV. C) There are new ways of protecting yourself from HIV that are being discovered every day. D) Other than abstinence, only the consistent and correct use of condoms is effective in preventing HIV.

Ans: D Feedback: Other than abstinence, consistent and correct use of condoms is the only effective method to decrease the risk of sexual transmission of HIV infection. Both female and male condoms confer significant protection. New prevention techniques are not commonly discovered, though advances in treatment are constant.

A patient has come into the free clinic asking to be tested for HIV infection. The patient asks the nurse how the test works. The nurse responds that if the testing shows that antibodies to the AIDS virus are present in the blood, this indicates what? A) The patient is immune to HIV. B) The patient's immune system is intact. C) The patient has AIDS-related complications. D) The patient has been infected with HIV.

Ans: D Feedback: Positive test results indicate that antibodies to the AIDS virus are present in the blood. The presence of antibodies does not imply an intact immune system or specific immunity to HIV. This finding does not indicate the presence of AIDS-related complications.

The nurse is addressing condom use in the context of a health promotion workshop. When discussing the correct use of condoms, what should the nurse tell the attendees? A Attach the condom prior to erection. B) A condom may be reused with the same partner if ejaculation has not occurred. C) Use skin lotion as a lubricant if alternatives are unavailable. D) Hold the condom by the cuff upon withdrawal.

Ans: D Feedback: The condom should be unrolled over the hard penis before any kind of sex. The condom should be held by the tip to squeeze out air. Skin lotions, baby oil, petroleum jelly, or cold cream should not be used with condoms because they cause latex deterioration/condom breakage. The condom should be held during withdrawal so it does not come off the penis. Condoms should never be reused.

During the admission assessment of an HIV-positive patient whose CD4+ count has recently fallen, the nurse carefully assesses for signs and symptoms related to opportunistic infections. What is the most common life-threatening infection? A) Salmonella infection B) Mycobacterium tuberculosis C) Clostridium difficile D) Pneumocystis pneumonia

Ans: D Feedback: There are a number of opportunistic infections that can infect individuals with AIDS. The most common life-threatening infection in those living with AIDS is Pneumocystis pneumonia (PCP), caused by P. jiroveci (formerly carinii). Other opportunistic infections may involve Salmonella,Mycobacterium tuberculosis, and Clostridium difficile.

The nurse should recognize a patient's risk for impaired immune function if the patient has undergone surgical removal of which of the following? A) Thyroid gland B) Spleen C) Kidney D) Pancreas

B (Feedback: A history of surgical removal of the spleen, lymph nodes, or thymus may place the patient at risk for impaired immune function. Removal of the thyroid, kidney, or pancreas would not directly lead to impairment of the immune system.)

A nurse has administered a child's scheduled vaccination for rubella. This vaccination will cause the child to develop which of the following? A) Natural immunity B) Active acquired immunity C) Cellular immunity D) Mild hypersensitivity

B (Feedback: Active acquired immunity usually develops as a result of vaccination or contracting a disease. Natural immunity is present at birth and provides a nonspecific response to any foreign invader. Immunizations do not activate the process of cellular immunity. Hypersensitivity is not an expected outcome of immunization.)

A nurse is explaining the process by which the body removes cells from circulation after they have performed their physiologic function. The nurse is describing what process? A) The cellular immune response B) Apoptosis C) Phagocytosis D) Opsonization

B (Feedback: Apoptosis, or programmed cell death, is the body's way of destroying worn out cells such as blood or skin cells or cells that need to be renewed. Opsonization is the coating of antigen-antibody molecules with a sticky substance to facilitate phagocytosis. The body does not use phagocytosis or the cellular immune response to remove cells from circulation.)

A patient is undergoing testing to determine the overall function of her immune system. What test can be performed to evaluate the functioning of the patient's cellular immune system? A) Immunoglobulin testing B) Delayed hypersensitivity skin test C) Specific antibody response D) Total serum globulin assessment

B (Feedback: Cellular (cell-mediated) immunity tests include the delayed hypersensitivity skin test, since this immune response is specifically dependent on the cellular immune response. Each of the other listed tests assesses functioning of the humoral immune system.)

A patient was recently exposed to infectious microorganisms and many T lymphocytes are now differentiating into killer T cells. This process characterizes what stage of the immune response? A) Effector B) Proliferation C) Response D) Recognition

B (Feedback: In the proliferation stage, T lymphocytes differentiate into cytotoxic (or killer) T cells, whereas B lymphocytes produce and release antibodies. This does not occur in the response, recognition, or effector stages.)

A 16-year-old has been brought to the emergency department by his parents after falling through the glass of a patio door, suffering a laceration. The nurse caring for this patient knows that the site of the injury will have an invasion of what? A) Interferons B) Phagocytic cells C) Apoptosis D) Cytokines

B (Feedback: Monocytes migrate to injury sites and function as phagocytic cells, engulfing, ingesting, and destroying greater numbers and quantities of foreign bodies or toxins than granulocytes. This occurs in response to the foreign bodies that have invaded the laceration from the dirt on the broken glass. Interferon, one type of biologic response modifier, is a nonspecific viricidal protein that is naturally produced by the body and is capable of activating other components of the immune system. Apoptosis, or programmed cell death, is the body's way of destroying worn out cells such as blood or skin cells or cells that need to be renewed. Cytokines are the various proteins that mediate the immune response. These do not migrate to injury sites.)

The nurse is assessing a client's risk for impaired immune function. What assessment finding should the nurse identify as a risk factor for decreased immunity? A) The patient takes a beta blocker for the treatment of hypertension. B) The patient is under significant psychosocial stress. C) The patient had a pulmonary embolism 18 months ago. D) The patient has a family history of breast cancer.

B (Feedback: Stress is a psychoneuroimmunologic factor that is known to depress the immune response. Use of beta blockers, a family history of cancer, and a prior PE are significant assessment findings, but none represents an immediate threat to immune function.)

A nurse is explaining how the humoral and cellular immune responses should be seen as interacting parts of the broader immune system rather than as independent and unrelated processes. What aspect of immune function best demonstrates this? A) The movement of B cells in and out of lymph nodes B) The interactions that occur between T cells and B cells C) The differentiation between different types of T cells D) The universal role of the complement system

B (Feedback: T cells interact closely with B cells, indicating that humoral and cellular immune responses are not separate, unrelated processes, but rather branches of the immune response that interact. Movement of B cells does not clearly show the presence of a unified immune system. The differentiation between types of T cells and the role of the complement system do not directly suggest a single immune system.)

A man was scratched by an old tool and developed a virulent staphylococcus infection. In the course of the man's immune response, circulating lymphocytes containing the antigenic message returned to the nearest lymph node. During what stage of the immune response did this occur? A) Recognition stage B) Proliferation stage C) Response stage D) Effector stage

B (Feedback: The recognition stage of antigens as foreign by the immune system is the initiating event in any immune response. The body must first recognize invaders as foreign before it can react to them. In the proliferation stage, the circulating lymphocyte containing the antigenic message returns to the nearest lymph node. Once in the node, the sensitized lymphocyte stimulates some of the resident dormant T and B lymphocytes to enlarge, divide, and proliferate. In the response stage, the differentiated lymphocytes function either in a humoral or a cellular capacity. In the effector stage, either the antibody of the humoral response or the cytotoxic (killer) T cell of the cellular response reaches and connects with the antigen on the surface of the foreign invader.)

A patient is admitted with cellulitis and experiences a consequent increase in white blood cell count. The nurse is aware that during the immune response, pathogens are engulfed by white blood cells that ingest foreign particles. What is this process known as? A) Apoptosis B) Phagocytosis C) Antibody response D) Cellular immune response

B (Feedback: During the first mechanism of defense, white blood cells, which have the ability to ingest foreign particles, move to the point of attack, where they engulf and destroy the invading agents. This is known as phagocytosis. The action described is not apoptosis (programmed cell death) or an antibody response. Phagocytosis occurs in the context of the cellular immune response, but it does not constitute the entire cellular response.)

A nurse knows of several patients who have achieved adequate control of their allergy symptoms using over-the-counter antihistamines. Antihistamines would be contraindicated in the care of which patient? A) A patient who has previously been treated for tuberculosis B) A pregnant woman at 30 weeks' gestation C) A patient who is on estrogen-replacement therapy D) A patient with a severe allergy to eggs

B) A pregnant woman at 30 weeks' gestation Antihistamines are contraindicated during the third trimester of pregnancy. Previous tuberculosis, hormone therapy, and food allergies do not contraindicate the use of antihistamines.

A child has been transported to the emergency department (ED) after a severe allergic reaction. The ED nurse is evaluating the patient's respiratory status. How should the nurse evaluate the patient's respiratory status? Select all that apply. A) Facilitate lung function testing. B) Assess breath sounds. C) Measure the child's oxygen saturation by oximeter. D) Monitor the child's respiratory pattern. E) Assess the child's respiratory rate.

B) Assess breath sounds. C) Measure the child's oxygen saturation by oximeter. D) Monitor the child's respiratory pattern. E) Assess the child's respiratory rate. The respiratory status is evaluated by monitoring the respiratory rate and pattern and by assessing for breathing difficulties, low oxygen saturation, or abnormal lung sounds such as wheezing. Lung function testing is a lengthy procedure that is not appropriate in an emergency context.

A nurse is performing the initial assessment of a patient who has a recent diagnosis of systemic lupus erythematosus (SLE). What skin manifestation would the nurse expect to observe on inspection? A) Petechiae B) Butterfly rash C) Jaundice D) Skin sloughing

B) Butterfly rash An acute cutaneous lesion consisting of a butterfly-shaped rash across the bridge of the nose and cheeks occurs in SLE. Petechiae are pinpoint skin hemorrhages,which are not a clinical manifestation of SLE. Patients with SLE do not typically experience jaundice or skin sloughing.

The nurse in an allergy clinic is educating a new patient about the pathology of the patient's health problem. What response should the nurse describe as a possible consequence of histamine release? A) Constriction of small venules B) Contraction of bronchial smooth muscle C) Dilation of large blood vessels D) Decreased secretions from gastric and mucosal cells

B) Contraction of bronchial smooth muscle Histamine's effects during the immune response include contraction of bronchial smooth muscle, resulting in wheezing and bronchospasm, dilation of small venules, constriction of large blood vessels, and an increase in secretion of gastric and mucosal cells.

A patient is receiving a transfusion of packed red blood cells. Shortly after initiation of the transfusion, the patient begins to exhibit signs and symptoms of a transfusion reaction. The patient is suffering from which type of hypersensitivity? A) Anaphylactic (type 1) B) Cytotoxic (type II) C) Immunecomplex (type III) D) Delayed type (type IV)

B) Cytotoxic (type II) A type II hypersensitivity reaction resulting in red blood cell destruction is associated with blood transfusions. This type of reaction does not result from types I, III, or IV reactions.

A nurse is caring for a patient who is suspected of having giant cell arteritis (GCA). What laboratory tests are most useful in diagnosing this rheumatic disorder? Select all that apply. A) Erythrocyte count B) Erythrocyte sedimentation rate C) Creatinine clearance D) C-reactive protein E) D-dimer

B) Erythrocyte sedimentation rate D) C-reactive protein Simultaneous elevation in the ESR and CRP have a sensitivity of 88% and a specificity of 98% in making the diagnosis of GCA when coupled with clinical findings. Erythrocyte counts, creatinine clearance, and D-dimer are not diagnostically useful.)

A patient with polymyositisis experiencing challenges with activities of daily living as a result of proximal muscle weakness. What is the most appropriate nursing action? A) Initiate a program of passive range of motion exercises B) Facilitate referrals to occupational and physical therapy C) Administer skeletal muscle relaxants as ordered D) Encourage a progressive program of weight-bearing exercise

B) Facilitate referrals to occupational and physical therapy (Patients with polymyositis may have symptoms similar to those of other inflammatory diseases. However, proximal muscle weakness is characteristic, making activities such as hair combing, reaching overhead, and using stairs difficult. Therefore, use of assistive devices may be recommended, and referral to occupational or physical therapy may be warranted. The muscle weakness is a product of the disease process, not lack of exercise. Skeletal muscle relaxants are not used in the treatment of polymyositis.)

A patient has just been told by his physician that he has scleroderma. The physician tells the patient that he is going to order some tests to assess for systemic involvement. The nurse knows that priority systems to be assessed include what? A) Hepatic B) Gastrointestinal C) Genitourinary D) Neurologic

B) Gastrointestinal Assessment of systemic involvement with scleroderma requires a systems review with special attention to gastrointestinal, pulmonary, renal, and cardiac systems. Liver, GU, and neurologic functions are not central priorities

A patient with rheumatic disease is complaining of stomatitis. The nurse caring for the patient should further assess the patient for the adverse effects of what medications? A) Corticosteroids B) Gold-containing compounds C) Antimalarials D) Salicylate therapy

B) Gold-containing compounds Stomatitis is an adverse effect that is associated with gold therapy. Steroids, antimalarials, and salicylates do not normally have this adverse effect.)

A patient has a documented history of allergies presents to the clinic. She states that she is frustrated by her chronic nasal congestion, anosmia (inability to smell) and inability to concentrate. The nurse should identify which of the following nursing diagnoses? A) Deficient Knowledge of Self-Care Practices Related to Allergies B) Ineffective Individual Coping with Chronicity of Condition and Need for Environmental Modification C) Acute Confusion Related to Cognitive Effects of Allergic Rhinitis D) Disturbed Body Image Related to Sequelae of Allergic Rhinitis

B) Ineffective Individual Coping with Chronicity of Condition and Need for Environmental Modification The most appropriate nursing diagnosis is Ineffective Individual Coping with Chronicity of Condition and Need for Environmental Modification. This nursing diagnosis is all encompassing of the subjective and objective data. Altered body image and acute confusion are not evidenced by the data. The patient's condition is not necessary attributable to a knowledge deficit.

A nurse is providing care for a patient who has just been diagnosed as being in the early stage of rheumatoid arthritis. The nurse should anticipate the administration of which of the following? A) Hydromorphone (Dilaudid) B) Methotrexate (Rheumatrex) C) Allopurinol (Zyloprim) D) Prednisone

B) Methotrexate (Rheumatrex) In the past, a step-wise approach starting with NSAIDs was standard of care. However, evidence clearly documenting the benefits of early DMARD(methotrexate[Rheumatrex], antimalarials, leflunomide [Arava], or sulfasalazine [Azulfidine]) treatment has changed national guidelines for management. Now it is recommended that treatment with the non-biologic DMARDs begin within 3 months of disease onset. Allopurinol is used to treat gout. Opioids are not indicated in early RA. Prednisone is used in unremitting RA.

A patient is learning about his new diagnosis of asthma with the asthma nurse. What medication has the ability to prevent the onset of acute asthma exacerbations? A) Diphenhydramine (Benadryl) B) Montelukast (Singulair) C) Albuterol sulfate (Ventolin) D) Epinephrine

B) Montelukast (Singulair) Many manifestations of inflammation can be attributed in part to leukotrienes. Medications categorized as leukotriene antagonists or modifiers such as montelukast (Singulair) block the synthesis or action of leukotrienes and prevent signs and symptoms associated with asthma. Diphenhydramine prevents histamine's effect on smooth muscle. Albuterol sulfate relaxes smooth muscle during an asthma attack. Epinephrine relaxes bronchial smooth muscle but is not used on a preventative basis.

A patient is brought to the emergency department (ED) in a state of anaphylaxis. What is the ED nurse's priority for care? A) Monitor the patient's level of consciousness. B) Protect the patient's airway. C) Provide psychosocial support. D) Administer medications as ordered.

B) Protect the patient's airway. Anaphylaxis severely threatens a patient's airway; the nurse's priority is preserving airway patency and breathing pattern. This is a higher priority than other valid aspects of care, including medication administration, psychosocial support, and assessment of LOC.

A patient was prescribed an oral antibiotic for the treatment of sinusitis. The patient has now stopped, stating she developed a rash shortly after taking the first dose of the drug. What is the nurse's most appropriate response? A) Encourage the woman to continue with the medication while monitoring her skin condition closely. B) Refer the woman to her primary care provider to have the medication changed. C) Arrange for the woman to go to the nearest emergency department. D) Encourage the woman to take an OTC antihistamine with each dose of the antibiotic.

B) Refer the woman to her primary care provider to have the medication changed. On discovery of a medication allergy, patients are warned that they have a hypersensitivity to a particular medication and are advised not to take it again. As a result, the patient would need to liaise with the primary care provider. There is no need for emergency care unless symptoms worsen to involve respiratory function. An antihistamine would not be an adequate or appropriate recommendation from the nurse.

A patient with rheumatoid arthritis comes to the clinic complaining of pain in the joint of his right great toe and is eventually diagnosed with gout. When planning teaching for this patient, what management technique should the nurse emphasize? A) Take OTC calcium supplements consistently. B) Restrict consumption of foods high in purines. C) Ensure fluid intake of at least 4 liters per day. D) Restrict weight-bearing on right foot.

B) Restrict consumption of foods high in purines. Although severe dietary restriction is not necessary, the nurse should encourage the patient to restrict consumption of foods high in purines, especially organ meats. Calcium supplementation is not necessary and activity should be maintained as tolerated. Increased fluid Intake is beneficial, but it is not necessary for the patient to consume more than 4 liters daily

A nurse is providing care for a patient who has a rheumatic disorder. The nurses comprehensive assessment includes the patients mood, behavior, LOC, and neurologic status. What is this patients most likely diagnosis? A) Osteoarthritis (OA) B) Systemic lupus erythematosus (SLE) C) Rheumatoid arthritis (RA) D) Gout

B) Systemic lupus erythematosus (SLE) SLE has a high degree of neurologic involvement, and can result in central nervous system changes. The patient and family members are asked about any behavioral changes, including manifestations of neurosis or psychosis. Signs of depression are noted, as are reports of seizures, chorea, or other central nervous system manifestations. OA, RA, and gout lack this dimension.

A 21-year-old male has just been diagnosed with a spondyloarthropathy. What will be a priority nursing intervention for this patient? A) Referral for assistive devices B) Teaching about symptom management C) Referral to classes to stop smoking D) Setting up an exercise program

B) Teaching about symptom management (Major nursing interventions in the spondyloarthropathies are related to symptom management and maintenance of optimal functioning. This is a priority over the use of assistive devices, smoking cessation, and exercise programs, though these topics may be of importance for some patients.)

A nurse at an allergy clinic is providing education for a patient starting immunotherapy for the treatment of allergies. What education should the nurse prioritize? A) The importance of scheduling appointments for the same time each month B) The importance of keeping appointments for desensitization procedures C) The importance of avoiding antihistamines for the duration of treatment D) The importance of keeping a diary of reactions to the immunotherapy

B) The importance of keeping appointments for desensitization procedures The nurse informs and reminds the patient of the importance of keeping appointments for desensitization procedures, because dosages are usually adjusted on a weekly basis, and missed appointments may interfere with the dosage adjustment. Appointments are more frequent than monthly and antihistamines are not contraindicated. There is no need to keep a diary of reactions.

A child has been diagnosed with a severe walnut allergy after suffering an anaphylactic reaction. What is a priority for health education? A) The need to begin immunotherapy as soon as possible B) The need for the parents to carry an epinephrine pen C) The need to vigilantly maintain the child's immunization status D) The need for the child to avoid all foods that have a high potential for allergies

B) The need for the parents to carry an epinephrine pen All patients with food allergies, especially seafood and nuts, should have an EpiPen device prescribed. The child does not necessarily need to avoid all common food allergens. Immunotherapy is not indicated in the treatment of childhood food allergies. Immunizations are important, but do not address food allergies.

A patient with severe environmental allergies is scheduled for an immunotherapy injection. What should be included in teaching the patient about this treatment? A) The patient will be given a low dose of epinephrine before the treatment. B) The patient will remain in the clinic to be monitored for 30 minutes following the injection. C) Therapeutic failure occurs if the symptoms to the allergen do not decrease after 3 months. D) The allergen will be administered by the peripheral intravenous route.

B) The patient will remain in the clinic to be monitored for 30 minutes following the injection. Although severe systemic reactions are rare, the risk of systemic and potentially fatal anaphylaxis exists. Because of this risk, the patient must remain in the office or clinic for at least 30 minutes after the injection and is observed for possible systemic symptoms. Therapeutic failure is evident when a patient does not experience a decrease in symptoms within 12 to 24 months. Epinephrine is not given prior to treatment and the IV route is not used.

A patient's decline in respiratory and renal function has been attributed to Goodpasture syndrome, which is a type II hypersensitivity reaction. What pathologic process underlies the patient's health problem? A) Antigens have bound to antibodies and formed inappropriate immune complexes. B) The patient's body has mistakenly identified a normal constituent of the body as foreign. C) Sensitized T cells have caused cell and tissue damage. D) Mast cells have released histamines that directly cause cell lysis.

B) The patient's body has mistakenly identified a normal constituent of the body as foreign. Type II reactions, or cytotoxic hypersensitivity, occur when the system mistakenly identifies a normal constituent of the body as foreign. An example of this type of reaction is Goodpasture syndrome. Type III, or immune complex, hypersensitivity involves immune complexes that are formed when antigens bind to antibodies. Type IV hypersensitivity is mediated by sensitized T cells that cause cell and tissue damage. Histamine does not directly cause cell lysis.

A nurse is assessing a patient for risk factors known to contribute to osteoarthritis. What assessment finding would the nurse interpret as a risk factor? A) The patient has a 30 pack-year smoking history. B) The patients body mass index is 34 (obese). C) The patient has primary hypertension. D) The patient is 58 years old.

B) The patients body mass index is 34 (obese). (Feedback:Risk factors for osteoarthritis include obesity and previous joint damage. Risk factors of OA do not include smoking or hypertension. Incidence increases with age, but a patient who is 58 would not yet face a significantly heightened risk.)

A nurse is caring for a teenage girl who has had an anaphylactic reaction after a bee sting. The nurse is providing patient teaching prior to the patient's discharge. In the event of an anaphylactic reaction, the nurse informs the patient that she should self-administer epinephrine in what site? A) Forearm B) Thigh C) Deltoid muscle D) Abdomen

B) Thigh The patient is taught to position the device at the middle portion of the thigh and push the device into the thigh as far as possible. The device will autoinject a premeasured dose of epinephrine into the subcutaneous tissue.

A nurse is planning patient education for a patient being discharged home with a diagnosis of rheumatoid arthritis. The patient has been prescribed antimalarials for treatment, so the nurse knows to teach the patient to self-monitor for what adverse effect? A) Tinnitus B) Visual changes C) Stomatitis D) Hirsutism

B) Visual changes Antimalarials may cause visual changes; regular ophthalmologic examinations are necessary.Tinnitus is associated with salicylate therapy, stomatitis is associated with gold therapy, and hirsutism is associated with corticosteroid therapy.)

A patient's recent diagnostic testing included a total lymphocyte count. The results of this test will allow the care team to gauge what aspect of the patient's immunity? A) Humoral immune function B) Antigen recognition C) Cell-mediated immune function D) Antibody production

C (Feedback: A total lymphocyte count is a test used to determine cellular immune function. It is not normally used for testing humoral immune function and the associated antigen-antibody.)

A gerontologic nurse is caring for an older adult patient who has a diagnosis of pneumonia. What age-related change increases older adults' susceptibility to respiratory infections? A) Atrophy of the thymus B) Bronchial stenosis C) Impaired ciliary action D) Decreased diaphragmatic muscle tone

C (Feedback: As a consequence of impaired ciliary action due to exposure to smoke and environmental toxins, older adults are vulnerable to lung infections. This vulnerability is not the result of thymus atrophy, stenosis of the bronchi, or loss of diaphragmatic muscle tone.)

The nurse knows that the response of natural immunity is enhanced by processes that are inherent in the physical and chemical barriers of the body. What is a chemical barrier that enhances the response of natural immunity? A) Cell cytoplasm B) Interstitial fluid C) Gastric secretions D) Cerebrospinal fluid

C (Feedback: Chemical barriers, such as mucus, acidic gastric secretions, enzymes in tears and saliva, and substances in sebaceous and sweat secretions, act in a nonspecific way to destroy invading bacteria and fungi. Not all body fluids are chemical barriers, however. Cell cytoplasm, interstitial fluid, and CSF are not normally categorized as chemical barriers to infection.)

A nurse is reviewing a patient's medication administration record in an effort to identify drugs that may contribute to the patient's recent immunosuppression. What drug is most likely to have this effect? A) An antibiotic B) A nonsteroidal anti-inflammatory drug (NSAID) C) An antineoplastic D) An antiretroviral

C (Feedback: Chemotherapy affects bone marrow function, destroying cells that contribute to an effective immune response and resulting in immunosuppression. Antibiotics in large doses cause bone marrow suppression, but antineoplastic drugs have the most pronounced immunosuppressive effect. NSAIDs and antiretrovirals do not normally have this effect.)

Diagnostic testing has revealed a deficiency in the function of a patient's complement system. This patient is likely to have an impaired ability to do which of the following? A) Protecting the body against viral infection B) Marking the parameters of the immune response C) Bridging natural and acquired immunity D) Collecting immune complexes during inflammation

C (Feedback: Complement has three major physiologic functions: defending the body against bacterial infection, bridging natural and acquired immunity, and disposing of immune complexes and the byproducts associated with inflammation. Complement does not mark the parameters of the immune response; complement does not collect immune complexes during inflammation.)

A patient's current immune response involves the direct destruction of foreign microorganisms. This aspect of the immune response may be performed by what cells? A) Suppressor T cells B) Memory T cells C) Cytotoxic T cells D) Complement T cells

C (Feedback: Cytotoxic T cells (also called CD8 + cells) participate in the destruction of foreign organisms. Memory T cells and suppressor T cells do not perform this role in the immune response. The complement system does not exist as a type of T cell.)

A nurse is admitting a patient who exhibits signs and symptoms of a nutritional deficit. Inadequate intake of what nutrient increases a patient's susceptibility to infection? A) Vitamin B12 B) Unsaturated fats C) Proteins D) Complex carbohydrates

C (Feedback: Depletion of protein reserves results in atrophy of lymphoid tissues, depression of antibody response, reduction in the number of circulating T cells, and impaired phagocytic function. As a result, the patient has an increased susceptibility to infection. Low intake of fat and vitamin B12 affects health, but is not noted to directly create a risk for infection. Low intake of complex carbohydrates is not noted to constitute a direct risk factor for infection.)

A patient is responding to a microbial invasion and the patient's differentiated lymphocytes have begun to function in either a humoral or a cellular capacity. During what stage of the immune response does this occur? A) The recognition stage B) The effector stage C) The response stage D) The proliferation stage

C (Feedback: In the response stage, the differentiated lymphocytes function in either a humoral or a cellular capacity. In the effector stage, either the antibody of the humoral response or the cytotoxic (killer) T cell of the cellular response reaches and connects with the antigen on the surface of the foreign invader. In the recognition stage, the recognition of antigens as foreign, or non-self, by the immune system is the initiating event in any immune response. During the proliferation stage the circulating lymphocytes containing the antigenic message return to the nearest lymph node.)

A nurse is planning the assessment of a patient who is exhibiting signs and symptoms of an autoimmune disorder. The nurse should be aware that the incidence and prevalence of autoimmune diseases is known to be higher among what group? A) Young adults B) Native Americans C) Women D) Hispanics

C (Feedback: Many autoimmune diseases have a higher incidence in females than in males, a phenomenon believed to be correlated with sex hormones.)

A patient with cystic fibrosis has received a double lung transplant and is now experiencing signs of rejection. What is the immune response that predominates in this situation? A) Humoral B) Nonspecific C) Cellular D) Mitigated

C (Feedback: Most immune responses to antigens involve both humoral and cellular responses, although only one predominates. During transplantation rejection, the cellular response predominates over the humoral response. Neither a mitigated nor nonspecific cell response is noted in this situation.)

A nurse is caring for a patient who has had a severe antigen/antibody reaction. The nurse knows that the portion of the antigen that is involved in binding with the antibody is called what? A) Antibody lock B) Antigenic sequence C) Antigenic determinant D) Antibody channel

C (Feedback: The portion of the antigen involved in binding with the antibody is referred to as the antigenic determinant. This portion is not known as an antibody lock, antigenic sequence, or antibody channel.)

A patient requires ongoing treatment and infection-control precautions because of an inherited deficit in immune function. The nurse should recognize that this patient most likely has what type of immune disorder? A) A primary immune deficiency B) A gammopathy C) An autoimmune disorder D) A rheumatic disorder

C (Feedback: Primary immune deficiency results from improper development of immune cells or tissues. These disorders are usually congenital or inherited. Autoimmune disorders are less likely to have a genetic component, though some have a genetic component. Overproduction of immunoglobulins is the hallmark of gammopathies. Rheumatic disorders do not normally involve impaired immune function)

A patient has been living with seasonal allergies for many years, but does not take antihistamines, stating, When I was young I used to take antihistamines, but they always put me to sleep. How should the nurse best respond? A) "Newer antihistamines are combined with a stimulant that offsets drowsiness." B) "Most people find that they develop a tolerance to sedation after a few months." C) "The newer antihistamines are different than in years past, and cause less sedation." D) "Have you considered taking them at bedtime instead of in the morning?"

C) "The newer antihistamines are different than in years past, and cause less sedation." Unlike first-generation H1 receptor antagonists, newer antihistamines bind to peripheral rather than central nervous system H1 receptors, causing less sedation, if at all. Tolerance to sedation did not usually occur with first-generation drugs and newer antihistamines are not combined with a stimulant.

A junior nursing student is having an observation day in the operating room. Early in the day, the student tells the OR nurse that her eyes are swelling and she is having trouble breathing. What should the nurse suspect? A) Cytotoxic reaction due to contact with the powder in the gloves B) Immune complex reaction due to contact with anesthetic gases C) Anaphylaxis due to a latex allergy D) Delayed reaction due to exposure to cleaning products

C) Anaphylaxis due to a latex allergy Immediate hypersensitivity to latex, a type I allergic reaction, is mediated by the IgE mast cell system. Symptoms can include rhinitis, conjunctivitis, asthma, and anaphylaxis. The term latex allergy is usually used to describe the type I reaction. The rapid onset is not consistent with a cytotoxic reaction, an immune complex reaction, or a delayed reaction.

A patient with rheumatoid arthritis comes into the clinic for a routine check-up. On assessment the nurse notes that the patient appears to have lost some of her ability to function since her last office visit. Which of the following is the most appropriate action? A) Arrange a family meeting in order to explore assisted living options. B) Refer the patient to a support group. C) Arrange for the patient to be assessed in her home environment. D) Refer the patient to social work.

C) Arrange for the patient to be assessed in her home environment. Assessment in the patients home setting can often reveal more meaningful data than an assessment in the health care setting. There is no indication that assisted living is a pressing need or that the patient would benefit from social work or a support group

A nurse is aware of the need to assess patients' risks for anaphylaxis. What health care procedure constitutes the highest risk for anaphylaxis? A) Administration of the measles-mumps-rubella (MMR) vaccine B) Rapid administration of intravenous fluids C) Computed tomography with contrast solution D) Administration of nebulized bronchodilators

C) Computed tomography with contrast solution Radiocontrast agents present a significant threat of anaphylaxis in the hospital setting. Vaccinations less often cause anaphylaxis. Bronchodilators and IV fluids are not implicated in hypersensitivity reactions.

A child is undergoing testing for food allergies after experiencing unexplained signs and symptoms of hypersensitivity. What food items would the nurse inform the parents are common allergens? A) Citrus fruits and rice B) Root vegetables and tomatoes C) Eggs and wheat D) Hard cheeses and vegetable oils

C) Eggs and wheat The most common causes of food allergies are seafood (lobster, shrimp, crab, clams, fish), legumes (peanuts, peas, beans, licorice), seeds (sesame, cottonseed, caraway, mustard, flaxseed, sunflower seeds), tree nuts, berries, egg white, buckwheat, milk, and chocolate.

A clinic nurse is caring for a patient diagnosed with rheumatoid arthritis (RA). The patient tells the nurse that she has not been taking her medication because she usually cannot remove the childproof medication lids. How can the nurse best facilitate the patients adherence to her medication regimen? A) Encourage the patient to store the bottles with their tops removed. B) Have a trusted family member take over the management of the patients medication regimen. C) Encourage her to have her pharmacy replace the tops with alternatives that are easier to open. D) Have the patient approach her primary care provider to explore medication alternatives.

C) Encourage her to have her pharmacy replace the tops with alternatives that are easier to open. The patients pharmacy will likely be able to facilitate a practical solution that preserves the patients independence while still Fostering adherence to treatment. There should be no need to change medications, and storing open medication containers is unsafe. Delegating medications to a Family member is likely unnecessary at this point and promotes dependence

A patient has presented with signs and symptoms that are consistent with contact dermatitis. What aspect of care should the nurse prioritize when working with this patient? A) Promoting adequate perfusion in affected regions B) Promoting safe use of topical antihistamines C) Identifying the offending agent, if possible D) Teaching the patient to safely use an EpiPen

C) Identifying the offending agent, if possible Identifying the offending agent is a priority in the care of a patient with dermatitis. Antihistamines are not administered topically and epinephrine is not used to treat dermatitis. Inadequate perfusion occurs with PAD or vasoconstriction.

A patient with systemic lupus erythematosus (SLE) is preparing for discharge. The nurse knows that the patient has understood health education when the patient makes what statement? A) Ill make sure I get enough exposure to sunlight to keep up my vitamin D levels. B) Ill try to be as physically active as possible between flare-ups. C) Ill make sure to monitor my body temperature on a regular basis. D) Ill stop taking my steroids when I get relief from my symptoms.

C) Ill make sure to monitor my body temperature on a regular basis. Fever can signal an exacerbation and should be reported to the physician. Sunlight and other sources of ultraviolet light may precipitate severe skin reactions and exacerbate the disease. Fatigue can cause a flare-up of SLE. Patients should be encouraged to pace activities and plan rest periods. Corticosteroids must be gradually tapered because they can suppress the function of the adrenal gland. As well, these drugs should not be independently adjusted by the patient.)

A clinic nurse is caring for a patient with suspected gout. While explaining the pathophysiology of gout to the patient, the nurse should describe which of the following? A) Autoimmune processes in the joints B) Chronic metabolic acidosis C) Increased uric acid levels D) Unstable serum calcium levels

C) Increased uric acid levels. Gout is caused by hyperuricemia (increased serum uric acid). Gout is not categorized as an autoimmune Disease and it does not result from metabolic acidosis or unstable serum calcium levels.

A patient with SLE has come to the clinic for a routine check-up. When auscultating the patients apical heart rate, the nurse notes the presence of a distinct scratching sound. What is the nurses most appropriate action? A) Reposition the patient and auscultate posteriorly. B) Document the presence of S3 and monitor the patient closely. C) Inform the primary care provider that a friction rub may be present. D) Inform the primary care provider that the patient may have pneumonia.

C) Inform the primary care provider that a friction rub may be present. Patients with SLE are susceptible to developing a pericardial friction rub, possibly associated with myocarditis and accompanying pleural effusions; this warrants prompt medical follow-up. This finding is not characteristic of pneumonia and does not constitute S3. Posterior auscultation is unlikely to yield additional meaningful data.)

A nurse is performing the health history and physical assessment of a patient who has a diagnosis of rheumatoid arthritis (RA). What assessment finding is most consistent with the clinical presentation of RA? A) Cool joints with decreased range of motion B) Signs of systemic infection C) Joint stiffness, especially in the morning D) Visible atrophy of the knee and shoulder joints

C) Joint stiffness, especially in the morning (In addition to joint pain and swelling, another classic sign of RA is joint stiffness, especially in the morning. Joints are typically swollen, not atrophied, and systemic infection does not accompany the disease. Joints are often warm rather than cool.)

A nurse is educating a patient with gout about lifestyle modifications that can help control the signs and symptoms of the disease. What recommendation should the nurse make? A) Ensuring adequate rest B) Limiting exposure to sunlight C) Limiting intake of alcohol D) Smoking cessation

C) Limiting intake of alcohol Alcohol and red meat can precipitate an acute exacerbation of gout. Each of the other listed actions is consistent with good health, but none directly addresses the factors that exacerbate gout.)

A nurse is creating a teaching plan for a patient who has a recent diagnosis of scleroderma. What topics should the nurse address during health education? Select all that apply. A) Surgical treatment options B) The importance of weight loss C) Managing Raynauds-type symptoms D) Smoking cessation E) The importance of vigilant skin care

C) Managing Raynauds-type symptoms D) Smoking cessation E) The importance of vigilant skin care Patient teaching for the patient with scleroderma focuses on management of Raynauds phenomenon, smoking cessation, and meticulous skin care. Surgical treatment options do not exist and weight loss is not a central concern.

A nurse has included the nursing diagnosis of Risk for Latex Allergy Response in a patient's plan of care. The presence of what chronic health problem would most likely prompt this diagnosis? A) Herpes simplex B) HIV C) Spina bifida D) Hypogammaglobulinemia

C) Spina bifida Patients with spina bifida are at a particularly high risk for developing a latex allergy. This is not true of patients with herpes simplex, HIV, or hypogammaglobulinemia.

A patient with SLE asks the nurse why she has to come to the office so often for check-ups. What would be the nurses best response? A) Taking care of you in the best way involves seeing you face to face. B) Taking care of you in the best way involves making sure you are taking your medication the way it is ordered. C) Taking care of you in the best way involves monitoring your disease activity and how well the prescribed treatment is working. D) Taking care of you in the best way involves drawing blood work every month.

C) Taking care of you in the best way involves monitoring your disease activity and how well the prescribed treatment is working. (The goals of treatment include preventing progressive loss of organ function, reducing the likelihood of acute disease, minimizing disease-related disabilities, and preventing complications from therapy. Management of SLE involves regular monitoring to assess disease activity and therapeutic effectiveness. Stating the benefit of face-to-face interaction does not answer the patient's question. Blood work is not necessarily drawn monthly and assessing medication adherence is not the sole purpose of visits.)

The nurse is completing a focused assessment addressing a patient's immune function. What should the nurse prioritize in the physical assessment? A) Percussion of the patient's abdomen B) Palpation of the patient's liver C) Auscultation of the patient's apical heart rate D) Palpation of the patient's lymph nodes

D (Feedback: During the assessment of immune function, the anterior and posterior cervical, supraclavicular, axillary, and inguinal lymph nodes are palpated for enlargement. If palpable nodes are detected, their location, size, consistency, and reports of tenderness on palpation are noted. Because of the central role of lymph nodes in the immune system, they are prioritized over the heart, liver, and abdomen, even though these would be assessed.)

A nurse is planning a patient's care and is relating it to normal immune response. During what stage of the immune response should the nurse know that antibodies or cytotoxic T cells combine and destroy the invading microbes? A) Recognition stage B) Proliferation stage C) Response stage D) Effector stage

D (Feedback: In the effector stage, either the antibody of the humoral response or the cytotoxic (killer) T cell of the cellular response reaches and couples with the antigen on the surface of the foreign invader. The coupling initiates a series of events that in most instances results in total destruction of the invading microbes or the complete neutralization of the toxin. This does not take place during the three preceding stages.)

A nurse has admitted a patient who has been diagnosed with urosepsis. What immune response predominates in sepsis? A) Mitigated B) Nonspecific C) Cellular D) Humoral

D (Feedback: Most immune responses to antigens involve both humoral and cellular responses, although only one predominates. For example, during transplantation rejection, the cellular response predominates, whereas in the bacterial pneumonias and sepsis, the humoral response plays the dominant role. Neither mitigated nor nonspecific cell response is noted in this situation.)

A neonate exhibited some preliminary signs of infection, but the infant's condition resolved spontaneously prior to discharge home from the hospital. This infant's recovery was most likely due to what type of immunity? A) Cytokine immunity B) Specific immunity C) Active acquired immunity D) Nonspecific immunity

D (Feedback: Natural immunity, or nonspecific immunity, is present at birth. Active acquired or specific immunity develops after birth. Cytokines are proteins that mediate the immune response; they are not a type of immunity.)

A patient's injury has initiated an immune response that involves inflammation. What are the first cells to arrive at a site of inflammation? A) Eosinophils B) Red blood cells C) Lymphocytes D) Neutrophils

D (Feedback: Neutrophils are the first cells to arrive at the site where inflammation occurs. Eosinophils increase in number during allergic reactions and stress responses, but are not always present during inflammation. RBCs do not migrate during an immune response. Lymphocytes become active but do not migrate to the site of inflammation.)

A nursing student is giving a report on the immune system. What function of cytokines should the student describe? A) Determining whether a cell is foreign B) Determining if lymphokines will be activated C) Determining whether the T cells will remain in the nodes and retain a memory of the antigen D) Determining whether the immune response will be the production of antibodies or a cell-mediated response

D (Feedback: Separate subpopulations of helper T cells produce different types of cytokines and determine whether the immune response will be the production of antibodies or a cell-mediated immune response. Cytokines do not determine whether cells are foreign, determine if lymphokines will be activated, or determine the role of memory T cells.)

A nurse is reviewing the immune system before planning an immunocompromised patient's care. How should the nurse characterize the humoral immune response? A) Specialized cells recognize and ingest cells that are recognized as foreign. B) T lymphocytes are assisted by cytokines to fight infection. C) Lymphocytesare stimulated to become cells that attack microbes directly. D) Antibodies are made by B lymphocytes in response to a specific antigen.

D (Feedback: The humoral response is characterized by the production of antibodies by B lymphocytes in response to a specific antigen. Phagocytosis and direct attack on microbes occur in the context of the cellular immune response.)

A patient is being treated for bacterial pneumonia. In the first stages of illness, the patient's dyspnea was accompanied by a high fever. Currently, the patient claims to be feeling better and is afebrile. The patient is most likely in which stage of the immune response? A) Recognition stage B) Proliferation stage C) Response stage D) Effector stage

D (Feedback: The immune response culminates with the effector stage, during which offending microorganisms are killed by the various actions of the immune system. The patient's improvement in health status is likely the result of this final stage in the immune response.)

A patient is being treated for cancer and the nurse has identified the nursing diagnosis of Risk for Infection Due to Protein Losses. Protein losses inhibit immune response in which of the following ways? A) Causing apoptosis of cytokines B) Increasing interferon production C) Causing CD4+ cells to mutate D) Depressing antibody response

D (Feedback: Depletion of protein reserves results in atrophy of lymphoid tissues, depression of antibody response, reduction in the number of circulating T cells, and impaired phagocytic function. This specific nutritional deficit does not cause T-cell mutation, an increase in the production of interferons, or apoptosis of cytokines.)

The nurse is providing health education to the parents of a toddler who has been diagnosed with food allergies. What should the nurse teach this family about the child's health problem? A) "Food allergies are a life-long condition, but most families adjust quite well to the necessary lifestyle changes." B) "Consistent use of over-the-counter antihistamines can often help a child overcome food allergies." C) "Make sure that you carry a steroid inhaler with you at all times, especially when you eat in restaurants." D) "Many children outgrow their food allergies in a few years if they avoid the offending foods."

D) "Many children outgrow their food allergies in a few years if they avoid the offending foods." Many food allergies disappear with time, particularly in children. About one-third of proven allergies disappear in 1 to 2 years if the patient carefully avoids the offending food. Antihistamines do not cure allergies and an EpiPen is carried, not a steroid inhaler.

Which of the following individuals would be the most appropriate candidate for immunotherapy? A) A patient who had an anaphylactic reaction to an insect sting B) A child with allergies to eggs and dairy C) A patient who has had a positive tuberculin skin test D) A patient with severe allergies to grass and tree pollen

D) A patient with severe allergies to grass and tree pollen The benefit of immunotherapy has been fairly well established in instances of allergic rhinitis and bronchial asthma that are clearly due to sensitivity to one of the common pollens, molds, or household dust. Immunotherapy is not used to treat type I hypersensitivities. A positive tuberculin skin test is not an indication for immunotherapy.

A patient is suspected of having rheumatoid arthritis and her diagnostic regimen includes aspiration of synovial fluid from the knee for a definitive diagnosis. The nurse knows that which of the following procedures will be involved? A) Angiography B) Myelography C) Paracentesis D) Arthocentesis

D) Arthocentesis Arthrocentesis involves needle aspiration of synovial fluid. Angiography is an x-ray study of circulation with a contrast agent injected into a selected artery. Myelography is an x-ray of the spinal subarachnoid space taken after the injection of a contrast agent into the spinal subarachnoid space through a lumbar puncture. Paracentesis is removal of fluid (ascites) from the peritoneal cavity through a small surgical incision or puncture made through the abdominal wall under sterile conditions

A nurse is providing care for a patient who has a recent diagnosis of giant cell arteritis (GCA). What aspect of physical assessment should the nurse prioritize? A) Assessment for subtle signs of bleeding disorders B) Assessment of the metatarsal joints and phalangeal joints C) Assessment for thoracic pain that is exacerbated by activity D) Assessment for headaches and jaw pain

D) Assessment for headaches and jaw pain Assessment of the patient with GCA focuses on musculoskeletal tenderness, weakness, and decreased function. Careful attention should be directed toward assessing the head (for changes in vision, headaches, and jaw claudication). There is not a particular clinical focus on the potential for bleeding, hand and foot pain, or thoracic pain.)

An adolescent patient's history of skin hyperreactivity and inflammation has been attributed to atopic dermatitis. The nurse should recognize that this patient consequently faces an increased risk of what health problem? A) Bronchitis B) Systemic lupus erythematosus (SLE) C) Rheumatoid arthritis D) Asthma

D) Asthma Nurses should be aware that atopic dermatitis is often the first step in a process that leads to asthma and allergic rhinitis. It is not linked as closely to bronchitis, SLE, and RA.

Allopurinol (Zyloprim) has been ordered for a patient receiving treatment for gout. The nurse caring for this patient knows to assess the patient for bone marrow suppression, which may be manifested by which of the following diagnostic findings? A) Hyperuricemia B) Increased erythrocyte sedimentation rate C) Elevated serum creatinine D) Decreased platelets

D) Decreased platelets Thrombocytopenia occurs in bone marrow suppression. Hyperuricemia occurs in gout, but is not caused by bone marrow suppression. Increased erythrocyte sedimentation rate may occur from inflammation associated with gout, but is not related to bone marrow suppression. An elevated serum creatinine level may indicate renal damage, but this is not associated with the use of allopurinol.)

A nurse is preparing a patient for allergy skin testing. Which of the following precautionary steps is most important for the nurse to follow? A) The patient must not have received an immunization within 7 days. B) The nurse should administer albuterol 30 to 45 minutes prior to the test. C) Prophylactic epinephrine should be administered before the test. D) Emergency equipment should be readily available.

D) Emergency equipment should be readily available. Emergency equipment must be readily available during testing to treat anaphylaxis. Immunizations do not contraindicate testing. Neither epinephrine nor albuterol is given prior to testing.

A nurse is assessing a patient with rheumatoid arthritis. The patient expresses his intent to pursue complementary and alternative therapies. What fact should underlie the nurses response to the patient? A) New evidence shows CAM to be as effective as medical treatment. B) CAM therapies negate many of the benefits of medications. C) CAM therapies typically do more harm than good. D) Evidence shows minimal benefits from most CAM therapies.

D) Evidence shows minimal benefits from most CAM therapies. A recent systematic review of complementary and alternative medicine (CAM) examined the efficacy of herbal medicine, acupuncture, Tai chi and biofeedback for the treatment of rheumatoid arthritis and osteoarthritis. Although acupuncture treatment for pain management showed some promise, in all modalities the evidence was ambiguous. There is not enough evidence of the effectiveness of CAM and more rigorous research is needed

A clinic nurse is caring for a patient newly diagnosed with fibromyalgia. When developing a care plan for this patient, what would be a priority nursing diagnosis for this patient? A) Impaired Urinary Elimination Related to Neuropathy B)Altered Nutrition Related to Impaired Absorption C) Disturbed Sleep Pattern Related to CNS Stimulation D) Fatigue Related to Pain

D) Fatigue Related to Pain (Feedback:Fibromyalgia is characterized by fatigue, generalized muscle aching, and stiffness. Impaired urinary elimination is not a common manifestation of the disease. Altered nutrition and disturbed sleep pattern are potential nursing diagnoses, but are not the priority.)

A patient with a family history of allergies has suffered an allergic response based on a genetic predisposition. This atopic response is usually mediated by what immunoglobulin? A) Immunoglobulin A B) Immunoglobulin M C) Immunoglobulin G D) Immunoglobulin E

D) Immunoglobulin E Atopy refers to allergic reactions characterized by the action of IgE antibodies and a genetic predisposition to allergic reactions.

The nurse is creating a care plan for a patient suffering from allergic rhinitis. Which of the following outcomes should the nurse identify? A) Appropriate use of prophylactic antibiotics B) Safe injection of corticosteroids C) Improved skin integrity D) Improved coping with lifestyle modifications

D) Improved coping with lifestyle modifications The goals for the patient with allergies may include restoration of normal breathing pattern, increased knowledge about the causes and control of allergic symptoms, improved coping with alterations and modifications, and absence of complications. Antibiotics are not used to treat allergies and corticosteroids, if needed, are not administered parenterally. Allergies do not normally threaten skin integrity.

A patient has developed severe contact dermatitis with burning, itching, cracking, and peeling of the skin on her hands. What should the nurse teach the patient to do? A) Wear powdered latex gloves when in public. B) Wash her hands with antibacterial soap every few hours. C) Maintain room temperature at 75°F to 80°F whenever possible. D) Keep her hands well-moisturized at all times.

D) Keep her hands well-moisturized at all times. Powdered latex gloves can cause contact dermatitis. Skin should be kept well-hydrated and should be washed with mild soap. Maintaining roomtemperature at 75°F to 80°F is not necessary.

A nurse is caring for a patient who has allergic rhinitis. What intervention would be most likely to help the patient meet the goal of improved breathing pattern? A) Teach the patient to take deep breaths and cough frequently. B) Use antihistamines daily throughout the year. C) Teach the patient to seek medical attention at the first sign of an allergic reaction. D) Modify the environment to reduce the severity of allergic symptoms.

D) Modify the environment to reduce the severity of allergic symptoms. The patient is instructed and assisted to modify the environment to reduce the severity of allergic symptoms or to prevent their occurrence. Deep breathing and coughing are not indicated unless an infection is present. Anaphylaxis requires prompt medical attention, but a minority of allergic reactions are anaphylaxis. Overuse of antihistamines reduces their effectiveness.

A nurses plan of care for a patient with rheumatoid arthritis includes several exercise-based interventions. Exercises for patients with rheumatoid disorders should have which of the following goals? A) Maximize range of motion while minimizing exertion B) Increase joint size and strength C) Limit energy output in order to preserve strength for healing D) Preserve and increase range of motion while limiting joint stress

D) Preserve and increase range of motion while limiting joint stress Exercise is vital to the management of rheumatic disorders. Goals should be preserving and promoting mobility and joint function while limiting stress on the joint and possible damage. Cardiovascular exertion should remain within age-based limits and individual ability, but it is not a goal to minimize exertion. Increasing joint size is not a valid goal

The nurse is providing care for a patient who has a diagnosis of hereditary angioedema. When planning this patient's care, what nursing diagnosis should be prioritized? A) Risk for Infection Related to Skin Sloughing B) Risk for Acute Pain Related to Loss of Skin Integrity C) Risk for Impaired Skin Integrity Related to Cutaneous Lesions D) Risk for Impaired Gas Exchange Related to Airway Obstruction

D) Risk for Impaired Gas Exchange Related to Airway Obstruction Edema of the respiratory tract can compromise the airway in patients with hereditary angioedema. As such, this is a priority nursing diagnosis over pain and possible infection. Skin integrity is not threatened by angioedema.

A 40-year-old woman was diagnosed with Raynauds phenomenon several years earlier and has sought care because of a progressive worsening of her symptoms. The patient also states that many of her skin surfaces are stiff, like the skin is being stretched from all directions. The nurse should recognize the need for medical referral for the assessment of what health problem? A) Giant cell arteritis (GCA) B) Fibromyalgia (FM) C) Rheumatoid arthritis (RA) D) Scleroderma

D) Scleroderma Scleroderma starts insidiously with Raynauds phenomenon and swelling in the hands. Later, the skin and the subcutaneous tissues become increasingly hard and rigid and cannot be pinched up from the underlying structures. This progression of symptoms is inconsistent with GCA, FM, or RA

A patients rheumatoid arthritis (RA) has failed to respond appreciably to first-line treatments and the primary care provider has added prednisone to the patients drug regimen. What principle will guide this aspect of the patients treatment? A) The patient will need daily blood testing for the duration of treatment. B) The patient must stop all other drugs 72 hours before starting prednisone. C) The drug should be used at the highest dose the patient can tolerate. D) The drug should be used for as short a time as possible.

D) The drug should be used for as short a time as possible. Corticosteroids are used for shortest duration and at lowest dose possible to minimize adverse effects. Daily blood work is not necessary and the patient does not need to stop other drugs prior to using corticosteroids.)

A patient who is scheduled for a skin test informs the nurse that he has been taking corticosteroids to help control his allergy symptoms. What nursing intervention should the nurse implement? A) The patient should take his corticosteroids regularly prior to testing. B) The patient should only be tested for grass, mold, and dust initially. C) The nurse should have an emergency cart available in case of anaphylaxis during the test. D) The patient's test should be cancelled until he is off his corticosteroids.

D) The patient's test should be cancelled until he is off his corticosteroids. Corticosteroids and antihistamines, including over-the-counter allergy medications, suppress skin test reactivity and should be stopped 48 to 96 hours before testing, depending on the duration of their activity. Emergency equipment must be at hand during allergy testing, but the test would be postponed.

A community health nurse is performing a visit to the home of a patient who has a history of rheumatoid arthritis (RA). On what aspect of the patients health should the nurse focus most closely during the visit? A) The patients understanding of rheumatoid arthritis B) The patients risk for cardiopulmonary complications C) The patients social support system D) The patients functional status

D) The patients functional status (The patient's functional status is a central focus of home assessment of the patient with RA. The nurse may also address the patient's understanding of the disease, complications, and social support, but the patient's level of function and quality of life is a primary concern.)

A patient is diagnosed with giant cell arteritis (GCA) and is placed on corticosteroids. A concern for this patient is that he will stop taking the medication as soon as he starts to feel better. Why must the nurseemphasize the need for continued adherence to the prescribed medication? A) To avoid complications such as venous thromboembolism B) To avoid the progression to osteoporosis C) To avoid the progression of GCA to degenerative joint disease D) To avoid complications such as blindness

D) To avoid complications such as blindness The nurse must emphasize to the patient the need for continued adherence to the prescribed medication regimen to avoid complications of giant cell arteritis, such as blindness. VTE, OP, and degenerative joint disease are not among the most common complications for GCA.)


Related study sets

Folders and Files - vocabulary & techniques

View Set

Muscle Excitation and Relaxation

View Set

Principles of Economics - Modules 4-7 Review

View Set

BASIC VEHICLE TECHNOLOGIES 2: ENGINES

View Set

Ch. 9 connect practice questions

View Set

Management of Patients with Immunodeficiency Disorders

View Set

chapter 25 intro to the health care

View Set

NUR 238 PrepU Chapter 1: Perspectives on Maternal and Child Health Care

View Set